2010-05-12 64 views
4

我想在两个对齐块之间对齐两个对齐字符,这样我可以在推导过程中使用维持水平对齐方程的一些文本。例如使用胶乳的以下摘录对齐LaTex,在对齐块之间对齐对齐字符

\begin{align*} 
    \frac{\delta \phi}{\delta x_1} = {} &\frac{9}{8}\frac{\delta_1\phi}{\delta_1x_1}-\frac{1}{8}\frac{\delta_3\phi}{\delta_3x_1} \\ 
    & \frac{9}{8}\frac{1}{h_1}\left[\phi(x_1+h_1/2)-\phi(x_i-h_1/2)\right]-\frac{1}{8}\frac{1}{3h_1}\left[\phi(x_i+3h_1/2)-\phi(x_1-3h_1/2)\right] 
\end{align*} 
some text in the middle 
\begin{align*} 
    & \frac{9}{8}\frac{1}{h_1}\left[\phi(x_1+h_1/2)-\phi(x_i-h_1/2)\right]-\frac{1}{8}\frac{1}{3h_1}\left[\phi(x_i+3h_1/2)-\phi(x_1-3h_1/2)\right] 
\end{align*} 

理想我想在第二块中的方程的左边与在第一块中的第二个方程的排队。我可以通过在中间没有文本来做一个解决方法,但是,我希望这个功能。

编辑

我想有文本之间一个良好的数额。假设三到四行与正常段落一致。在对齐块中添加文本是我很少提到的解决方法。

回答

6

使用\noalign

\begin{align*} 
    \frac{\delta \phi}{\delta x_1} = {} &\frac{9}{8}\frac{\delta_1\phi}{\delta_1x_1}-\frac{1}{8}\frac{\delta_3\phi}{\delta_3x_1} \\ 
    & \frac{9}{8}\frac{1}{h_1}\left[\phi(x_1+h_1/2)-\phi(x_i-h_1/2)\right]-\frac{1}{8}\frac{1}{3h_1}\left[\phi(x_i+3h_1/2)-\phi(x_1-3h_1/2)\right]\\ 
    \noalign{\noindent some text in the middle.} 
    & \frac{9}{8}\frac{1}{h_1}\left[\phi(x_1+h_1/2)-\phi(x_i-h_1/2)\right]-\frac{1}{8}\frac{1}{3h_1}\left[\phi(x_i+3h_1/2)-\phi(x_1-3h_1/2)\right] 
\end{align*} 
+0

这工作。谢谢 – ccook 2010-05-12 10:26:56

+2

我一直使用'\ intertext'来代替'\ noalign'。你知道两者是否有区别吗? – 2010-05-12 14:01:16

+1

'\ noalign'是内部的TeX命令。 '\ intertext'使用'\ noalign',它是'\ ifvmode \ else \\\ @ empty \ fi \ noalign {\ penalty \ postdisplaypenal \ vskip \ belowdisplayskip \ vbox {\ normalbaselines \ ifdim \ linewidth = \ columnwidt h \ else \ parshape \ @ne \ @totalleftmargin \ linewidth \ fi \ noindent#1 \ par} \ pena lty \ predisplaypenal \ vskip \ abovedisplayskip}' – 2010-05-12 14:15:01

2

尝试以下操作:

\begin{align*} 
    \frac{\delta \phi}{\delta x_1} = {} &\frac{9}{8}\frac{\delta_1\phi}{\delta_1x_1}-\frac{1}{8}\frac{\delta_3\phi}{\delta_3x_1} \\ 
    & \frac{9}{8}\frac{1}{h_1}\left[\phi(x_1+h_1/2)-\phi(x_i-h_1/2)\right]-\frac{1}{8}\frac{1}{3h_1}\left[\phi(x_i+3h_1/2)-\phi(x_1-3h_1/2)\right]\\ 
    & \mbox{some text in the middle} \\ 
    & \frac{9}{8}\frac{1}{h_1}\left[\phi(x_1+h_1/2)-\phi(x_i-h_1/2)\right]-\frac{1}{8}\frac{1}{3h_1}\left[\phi(x_i+3h_1/2)-\phi(x_1-3h_1/2)\right] 
\end{align*} 

如果你有大量的文字,你可能想使用\phantom

\begin{align*} 
    \frac{\delta \phi}{\delta x_1} = {} &\frac{9}{8}\frac{\delta_1\phi}{\delta_1x_1}-\frac{1}{8}\frac{\delta_3\phi}{\delta_3x_1} \\ 
    & \frac{9}{8}\frac{1}{h_1}\left[\phi(x_1+h_1/2)-\phi(x_i-h_1/2)\right]-\frac{1}{8}\frac{1}{3h_1}\left[\phi(x_i+3h_1/2)-\phi(x_1-3h_1/2)\right] 
\end{align*} 
Some text in the middle, more text, more text, more text, more text, more text, more text, more text, more text, more text, more text, more text, more text, more text, more text, more text, more text, more text, more text, more text, 
\begin{align*} 
    \phantom{\frac{\delta \phi}{\delta x_1} = {}} & \frac{9}{8}\frac{1}{h_1}\left[\phi(x_1+h_1/2)-\phi(x_i-h_1/2)\right]-\frac{1}{8}\frac{1}{3h_1}\left[\phi(x_i+3h_1/2)-\phi(x_1-3h_1/2)\right] 
\end{align*} 

如果你不想使用\phantom,唯一的解决办法我可以拿出如下:

\usepackage{multirow} 
.... 
\begin{document} 
\[ \begin{array}{rl} 
     \frac{\delta \phi}{\delta x_1} = {} &\frac{9}{8}\frac{\delta_1\phi}{\delta_1x_1}-\frac{1}{8}\frac{\delta_3\phi}{\delta_3x_1} \\ 
     & \frac{9}{8}\frac{1}{h_1}\left[\phi(x_1+h_1/2)-\phi(x_i-h_1/2)\right]-\frac{1}{8}\frac{1}{3h_1}\left[\phi(x_i+3h_1/2)-\phi(x_1-3h_1/2)\right]\\[5mm] 
\multicolumn{2}{l}{\parbox{\linewidth}{Some text in the middle, more text, more text, more text, more text, more text, more text, more text, more text, more text, more text, more text, more text, more text, more text, more text, more text, more text, more text, more text}}\\[1cm] 
     & \frac{9}{8}\frac{1}{h_1}\left[\phi(x_1+h_1/2)-\phi(x_i-h_1/2)\right]-\frac{1}{8}\frac{1}{3h_1}\left[\phi(x_i+3h_1/2)-\phi(x_1-3h_1/2)\right] 
\end{array} \] 
\end{document} 

我不认为有一个str “连接”一个表的一列宽度/ eqnarray /与另一个列对齐。

+0

这适用于短文本,但是如果文本较长,它不能正确包装。左对齐也不符合它周围的段落。非常感谢 – ccook 2010-05-12 09:18:26

+0

看到我编辑的答案。 – aioobe 2010-05-12 09:28:12

+0

谢谢,但是,有可能'链接'两个对齐字符?使用幻影需要我创建第一个等式中最广泛部分的幻影实例,不是吗?它也不会更新来自下部对齐块的上部对齐。或者这是不可能的? – ccook 2010-05-12 09:43:46

1

如果您不需要太多文本,则可以使用数学环境中的\text命令在单个align*环境中显示文本。

\begin{align*} 
    \frac{\delta \phi}{\delta x_1} = {} &\frac{9}{8}\frac{\delta_1\phi}{\delta_1x_1}-\frac{1}{8}\frac{\delta_3\phi}{\delta_3x_1} \\ 
    & \frac{9}{8}\frac{1}{h_1}\left[\phi(x_1+h_1/2)-\phi(x_i-h_1/2)\right]-\frac{1}{8}\frac{1}{3h_1}\left[\phi(x_i+3h_1/2)-\phi(x_1-3h_1/2)\right]\\ 
    \text{some text in the middle.}\\ 
    & \frac{9}{8}\frac{1}{h_1}\left[\phi(x_1+h_1/2)-\phi(x_i-h_1/2)\right]-\frac{1}{8}\frac{1}{3h_1}\left[\phi(x_i+3h_1/2)-\phi(x_1-3h_1/2)\right] 
\end{align*} 
+0

但是,谢谢你,问题是我希望有一大段文字,比如说一小段文字。 – ccook 2010-05-12 09:19:49

5

有一个非常方便的命令:

\begin{align*} 
    \int_0^1 x^2 &= \frac{1}{3} \\ 
    \intertext{I am the intertext. I am typesetted as normal text but dude, 
       the tabbing/alignment is carried over to the next slide. 
       Check it out!} 
    \int_0^1 x^3 &= \frac{1}{4} 
\end{align*} 

它在AMS描述userguide据我记得。但是,我最近才发现它。

+0

好的发现!谢谢! – ccook 2010-08-26 02:20:12